Storing Cookies (See : http://ec.europa.eu/ipg/basics/legal/cookies/index_en.htm ) help us to bring you our services at overunity.com . If you use this website and our services you declare yourself okay with using cookies .More Infos here:
https://overunity.com/5553/privacy-policy/
If you do not agree with storing cookies, please LEAVE this website now. From the 25th of May 2018, every existing user has to accept the GDPR agreement at first login. If a user is unwilling to accept the GDPR, he should email us and request to erase his account. Many thanks for your understanding

User Menu

Custom Search

Author Topic: Sum of torque  (Read 173221 times)

EOW

  • Sr. Member
  • ****
  • Posts: 383
Re: Sum of torque
« Reply #240 on: October 08, 2015, 10:03:44 AM »
I move in translation all the device. Like that I can have -X and +X for the velocity on the green part. The green part receive a torque. The arm don't receive a torque and the disk accelerates the rotation around itself.

EOW

  • Sr. Member
  • ****
  • Posts: 383
Re: Sum of torque
« Reply #241 on: October 09, 2015, 08:56:22 PM »
With the pressure with the law 1/d². The torque from the SideA is not the same than the torque from the SideB. I drawn only 2 springs but there are in all the blue surface.
« Last Edit: October 10, 2015, 12:18:16 AM by EOW »

EOW

  • Sr. Member
  • ****
  • Posts: 383
Re: Sum of torque
« Reply #242 on: October 10, 2015, 09:28:23 AM »
In the last image, all the black lines are link together. All the device I drawn turn around the red axis. Black lines "separate" the pressure from the top.

On the SideB the mean pressure is 0 if the number of black line is infinite. But with 100 lines for example the force is decrease by 100.

On the SideA, the torque is integrale(x²(x-R)dx) from 0 to R = R^4/4-R^3*R/3

With R=5, the torque on the SideB is hum maybe 5 and it is 52.0833 for the SideA.

There is no torque from the curved shape because the device turns around the red center and this is the center of the half circle.

There is no torque from straight horizontal lines and springs because they cancel themselves all torques IF the number of horizontal lines inside the half disk is high.

The attraction can be in another law than 1/d²

I added an image, N is the number of black lines. Here N=9 for example. At top in each slice the pressure is 0, at bottom the pressure is R/N.

I drawn the device after 30°, to look how the device rotates, all the device rotates.
« Last Edit: October 10, 2015, 08:29:25 PM by EOW »

EOW

  • Sr. Member
  • ****
  • Posts: 383
Re: Sum of torque
« Reply #243 on: October 10, 2015, 11:01:38 PM »
Image 1/ With half disk the sum of torque is 0

Image 2/ With a full disk. Springs are only in the right half disk. Springs push balls. The law of repulsion for springs is like d². The force is higher when the cos(angle) is lower so there is a difference.

Image 3/ With balls inside in PartB and balls outside in partA1/PartA2

Image4/ With water. F1>F2 so the disk want to turn clockwise. F3 gives a clockwise torque to the device around the red center. All the device is always in that position because I synchronized 2 angular velocities.
« Last Edit: October 11, 2015, 10:53:19 AM by EOW »

EOW

  • Sr. Member
  • ****
  • Posts: 383
Re: Sum of torque
« Reply #244 on: October 11, 2015, 11:51:21 AM »
I came back with this device. If I take a half disk, the centrifugal forces turns the half disk and the arm receives a torque. The arm turns at w (labo reference) and the disk turn around itself at w.cos(a) (labo reference) like that the disk is always in the position I draww.
« Last Edit: October 11, 2015, 08:37:32 PM by EOW »

EOW

  • Sr. Member
  • ****
  • Posts: 383
Re: Sum of torque
« Reply #245 on: October 12, 2015, 08:24:41 PM »
The calculations for the torques in the first image. Eq1 is the torque from the centrifugal force from the half disk itself around itself to the arm. Eq2 is the torque on the arm from the centrifugal force around the support. Eq3 is the torque on the disk. With:

d: the distance of the arm
alpha: the angle
R: the radius of the disk
w: the angular velocity of the arm

I can take 2 half disks like the image shows, like that Eq1=0 for 2 half disks. And like d1 different of d2 the torque on the arm + torque on the half disks are not at 0.


EOW

  • Sr. Member
  • ****
  • Posts: 383
Re: Sum of torque
« Reply #246 on: October 16, 2015, 07:03:46 PM »
The sum of torques is not 0.
« Last Edit: October 16, 2015, 09:21:46 PM by EOW »

EOW

  • Sr. Member
  • ****
  • Posts: 383
Re: Sum of torque
« Reply #247 on: October 17, 2015, 10:09:27 AM »
Better, if the center of the quarter circle of the white shape is the red center not the black center.

All the last device I explained are mounted on a device with an arm with an angle and a disk, I drawn only the disk, look at the third image.

EOW

  • Sr. Member
  • ****
  • Posts: 383
Re: Sum of torque
« Reply #248 on: October 18, 2015, 07:51:33 PM »
With the spring. The arm lost the energy FRwtsin(a)cos(a) and spring win the energy FRsin(wt.cos(a))sin(a), the sum of energy is not constant. It's like to compare:

b.cos(a) and sin(b*cos(a))

and the result is not the same, an example:

a=2.09
b=0.1

The result is -0.05 and -0.049979

Another example:

a=0.001
b=60*pi/180

The result is 1.74e-5 and 3.04e.7, the relation is 57.29 = 180/pi, it's the maximum of the relation

« Last Edit: October 18, 2015, 10:08:08 PM by EOW »

EOW

  • Sr. Member
  • ****
  • Posts: 383
Re: Sum of torque
« Reply #249 on: October 21, 2015, 04:00:49 PM »
At start, the disks don't turn around themselve in the lab reference. The arm turns at w. The spring gives the forces F1 and F2. Like the disks don't turn around themselves the lenght to the spring changes very few. But the forces F1b and F2b give a torque to the arm, this torque depends of w.

EOW

  • Sr. Member
  • ****
  • Posts: 383
Re: Sum of torque
« Reply #250 on: October 24, 2015, 08:35:48 PM »
With 2 different radius and a belt.

Or 2 disk with the same radius and a circle with friction to accelerate the disks
« Last Edit: October 25, 2015, 05:34:06 AM by EOW »

EOW

  • Sr. Member
  • ****
  • Posts: 383
Re: Sum of torque
« Reply #251 on: October 25, 2015, 10:58:33 AM »
The radius of the grey disk is lower than the red disk but the angle alpha of the grey disk is lower than the red disk, so the grey disk turns quickly than the red disk, the arm receives a clockwise torque.

EOW

  • Sr. Member
  • ****
  • Posts: 383
Re: Sum of torque
« Reply #252 on: October 26, 2015, 08:54:55 AM »
Like that the spring increases its potential energy and the arm receive a positive torque. The horizontal torus turns at -w in the arm reference. The disk turns at -0.707w in the arm reference.

EOW

  • Sr. Member
  • ****
  • Posts: 383
Re: Sum of torque
« Reply #253 on: October 27, 2015, 07:37:02 PM »
2 springs and 2 disks.
« Last Edit: October 28, 2015, 12:27:39 AM by EOW »

EOW

  • Sr. Member
  • ****
  • Posts: 383
Re: Sum of torque
« Reply #254 on: October 28, 2015, 01:14:27 PM »
On the red arm I can put 2 spirals. The spiral1 will accelerate counterclockwise in the arm reference and it is correct because R1 becomes smaller more and more and R2 becomes higher more and more. So, the spiral1 accelerates in the laboratory reference.The spiral2 decelerates in the arm reference but it accelerates in the laboratory reference. There is a pressure from a gas outside the device.